¿Por qué el campo eléctrico de un plano de carga infinitamente aislado es perpendicular al plano?

Estoy estudiando la ley de Gauss y encontré una sección en la que se supone que debemos encontrar el campo eléctrico de varias formas (como una línea infinita de cargas, etc.) y para un plano infinito con una carga positiva uniforme por área. , dice aquí en mis notas:

Simetría plana => La distribución de carga no cambia si la deslizamos en cualquier dirección paralela a la hoja => En cada punto, el campo es perpendicular a la hoja , y debe tener la misma magnitud a cualquier distancia dada a cada lado de la sábana.

No me queda claro por qué tener una distribución de carga que no cambia dará como resultado un campo perpendicular a la hoja. ¿Alguien puede ayudarme a aclarar?

Si la distribución de carga es uniforme, ¿qué otra dirección además de la perpendicular podría tomar un campo uniforme?
Creo que diría que podría haber un componente en el avión.
Relacionado: physics.stackexchange.com/q/441005/2451 y enlaces allí.

Respuestas (6)

La respuesta de @BrianMoths es correcta. Vale la pena aprender el idioma que se usa allí para ayudarlo con sus futuros estudios. Pero como introducción, aquí hay una explicación simplificada.

Comience con su distribución de carga y una "suposición" de la dirección del campo eléctrico.

ingrese la descripción de la imagen aquí

Como puede ver, hice que la conjetura tuviera un componente hacia arriba. Veremos en breve por qué esto conduce a una contradicción.

Ahora haz una "operación de simetría", que es una frase elegante para "hacer algo que deje algo sin cambiar". En este caso, voy a reflejar todo sobre una línea horizontal. Me refiero a todo .

ingrese la descripción de la imagen aquí

La "parte superior" de la hoja se convirtió en la "parte inferior". Esto es solo un etiquetado arbitrario, por lo que puede notar que invertí la distribución de carga. El campo eléctrico también se invierte. (Imagínate mirar todo en un espejo y te darás cuenta de por qué las cosas se invierten de la forma en que están).

Con suerte, todo está bien hasta ahora. Pero ahora compare la situación original con la nueva invertida.

ingrese la descripción de la imagen aquí

Tienes exactamente la misma distribución de carga. No se puede decir que lo volteé, excepto por mi etiquetado arbitrario. Pero si tiene la misma distribución de carga, también debe tener el mismo campo eléctrico. Como puede ver, este no es el caso, lo que significa que cometí un error en alguna parte.

La única dirección del campo eléctrico que no conduce a esta contradicción es la perpendicular a la lámina de carga.

Tienes que darte cuenta de que el sistema es invariante bajo rotaciones sobre la normal al plano. Entonces el campo eléctrico también debe ser invariante bajo estas rotaciones. Un componente de campo eléctrico en el plano cambia bajo tal rotación, por lo que tal componente no debe existir si tenemos esta invariancia. Así, el campo eléctrico es puramente a lo largo de la normal a la superficie.

Elija un punto por encima del plano.

Desde un punto en el plano directamente debajo del punto de arriba, dibuja un círculo de algún radio.

Considere la contribución de los elementos de carga a lo largo del círculo al campo eléctrico en el punto sobre el plano.

Dado que la densidad de carga es uniforme, las componentes horizontales del campo eléctrico de los elementos de carga en los lados opuestos del anillo se cancelan dejando solo las componentes verticales que se suman.

ingrese la descripción de la imagen aquí

Dado que esto es válido para todos estos anillos, el campo eléctrico en el punto por encima del plano debe ser perpendicular al plano de carga.

Una respuesta relacionada con la ley de Gauss (espero que todo sea correcto, ya que hace mucho tiempo para mí ... así que no hay garantía):

Un plano infinito de carga uniforme, por ejemplo en el plano z, tiene la distribución de carga:

ρ = q d ( z )

Por lo tanto, el potencial electrostático debe ser Φ = q | z | 2 π . Por tanto, el campo vectorial eléctrico es: mi = graduado ( Φ ) = { 0 , 0 , q firmar ( z ) 2 π }

En Mathematica puedo trazar esto como:

Show[ContourPlot3D[z == 0, {x, -1, 1}, {y, -1, 1}, {z, -0.5, 0.5}], VectorPlot3D[{0, 0, z/(2 \[Pi] (z^2)^(1/2))}, {x, -1, 1}, {y, -1, 1}, {z, -0.5, 0.5}]]

ingrese la descripción de la imagen aquí

Por supuesto, esto puede generalizarse. Espero que esto te ayude.

El problema es esencialmente unidimensional, por lo que debe usar la función de Green 1-D para la ecuación de Poisson, proporcional a | z | , no el 3D proporcional a | z | 1 . El potencial es lineal y la intensidad del campo constante (con un salto en la placa cambiando su dirección).
Tienes toda la razón, también pensé en esto ayer, pero me dio pereza resolverlo ... Cambiaré la respuesta en consecuencia.
... Espero que todos los signos sean correctos.

Con respecto a tu carga de prueba, siempre habrá el mismo número de cargas en tu plano en todas las direcciones porque el plano es infinito. Entonces, por cada cargo "en frente" de su cargo de prueba, habrá un cargo "detrás" de su cargo de prueba. Y por cada cargo a la izquierda de su cargo de prueba habrá un cargo a la derecha. Lo que esto significa es que no importa dónde coloque su carga de prueba, la fuerza paralela entre la carga de prueba y cualquier carga particular en el avión será cancelada por la fuerza paralela entre la carga de prueba y alguna otra carga particular en el avión. . Así es como yo lo pienso intuitivamente, al menos.

Asumir un componente en el plano del campo
Ahora tenemos que contradecir la existencia de este componente
Luego tomar un camino cerrado en forma de rectángulo que tiene dos de sus lados perpendiculares al plano y los lados que son paralelos al plano están en la dirección de la dirección asumida del componente paralelo del campo.
Ahora, el trabajo realizado en este ciclo cerrado no es cero.
Por lo tanto, no hay componente paralelo al plano.